subject
Mathematics, 08.05.2021 09:00 Carlosanddana123

Verify the following reduction formula: \displaystyle \int \sec^n(u)\, du=\frac{\sec^{n-2}(u)\tan(u)}{n-1}+\frac{n-2}{n-1}\int \sec^{n-2}(u)\, du, \; n\neq 1ā€‹

ansver
Answers: 2

Another question on Mathematics

question
Mathematics, 20.06.2019 18:04
What two consecutive odd integers have a sum of -28
Answers: 1
question
Mathematics, 21.06.2019 20:00
You wanted to draw an enlargement of design that printed on a card that is 4 in by 5
Answers: 1
question
Mathematics, 21.06.2019 22:30
If x-1 and x+3 are the factors of x3+10x2+ax+b find a and b
Answers: 1
question
Mathematics, 22.06.2019 02:10
Find the amount of simple interest earned for depositing the given principle in an account if $2200 is invested at 5.5 % for 6 months
Answers: 2
You know the right answer?
Verify the following reduction formula: ā€‹...
Questions
question
Mathematics, 11.09.2020 21:01
question
Mathematics, 11.09.2020 21:01
question
Mathematics, 11.09.2020 21:01
question
Mathematics, 11.09.2020 21:01
question
Mathematics, 11.09.2020 21:01
question
Social Studies, 11.09.2020 21:01
question
Mathematics, 11.09.2020 21:01
question
Mathematics, 11.09.2020 21:01
question
Mathematics, 11.09.2020 21:01
question
Mathematics, 11.09.2020 21:01
question
Biology, 11.09.2020 21:01
question
Mathematics, 11.09.2020 21:01
question
Mathematics, 11.09.2020 21:01
question
Mathematics, 11.09.2020 21:01
question
Mathematics, 11.09.2020 21:01
question
Mathematics, 11.09.2020 21:01
question
Mathematics, 11.09.2020 21:01
question
Mathematics, 11.09.2020 21:01
question
Spanish, 11.09.2020 21:01
question
Mathematics, 11.09.2020 21:01
Questions on the website: 13722367